Mechanical Circulatory Support



Mechanical Circulatory Support


William P. Mulvoy III





1. Which of the following echocardiography findings is not considered a contraindication prior to left ventricular assist device (LVAD) placement?


A. Bioprosthetic aortic valve with moderate aortic regurgitation


B. LV aneurysm


C. Moderate pulmonic stenosis


D. Right atrial appendage thrombus

View Answer

1. Correct Answer: D. Right atrial appendage thrombus

Rationale: Review of the American Society of Echocardiography Guidelines for LVAD management demonstrates multiple contraindications and/or red flags that need to be addressed prior to LVAD implantation. Table 36.1 from these guidelines discusses the echocardiography findings that must be addressed prior to separation from cardiopulmonary bypass and LVAD implantation. A bioprosthetic aortic valve with moderate aortic regurgitation needs to be repaired because once the LVAD begins to flow, the aortic regurgitation will get worse, causing a potential recirculation loop between the LVAD inflow cannula and the outflow cannula through the aortic valve. If there is an LV aneurysm, implantation of the inflow cannula may be hindered by poor tissue and/or difficulty with appropriate inflow cannula positioning. Pulmonic stenosis greater than mild should be addressed to prevent any impedance of right-sided blood flow once the LVAD is implanted. A right atrial appendage clot is not as important to address as a left atrial appendage thrombus, which could potentially embolize into the LVAD pump.

Selected Reference

1. Stainback RF, Estep JD, Agler DA, et al. Echocardiography in management of patients with left ventricular assist devices: recommendations from the American Society of Echocardiography. J Am Soc Echocardiogr. 2015;28(8):853-909.



2. A 48-year-old patient with dilated cardiomyopathy and a left ventricular ejection fraction (LVEF) of <10% presents for placement of a continuous flow LVAD as a bridge to cardiac transplantation. Upon separation from cardiopulmonary bypass, severe aortic insufficiency is visualized with no other valvular pathology. Calculate the aortic valve regurgitant blood flow given the following information:

Heart rate = 100 bpm







A. 2.4 L/min


B. 2.6 L/min


C. 2.2 L/min


D. 2.8 L/min

View Answer

2. Correct Answer: C. 2.2 L/min

Rationale/Critique: In this question the patient has two different ways that the LVAD receives blood flow. The first flow is from the right ventricular outflow tract (RVOT), thus the entire right-sided cardiac output is going into the left ventricle. The other flow into the LVAD is the regurgitant flow from the aortic valve regurgitation. To calculate the regurgitant blood flow, we need to use the following equations:

QLVAD = QRVOT + QAI > QAI = QLVAD – QRVOT

Stroke volume (SV) = 3.14 * r2 * VTI

Therefore,

QLVAD = SVLVAD * HR = 3.14 * (0.7)2 * 25.4 cm * 100 bpm = 3908 mL/min ˜ 3.9 L/min

QRVOT = SVRVOT * HR = 3.14 * (0.5)2 * 21.8 cm* 100 bpm = 1711 mL/min ˜ 1.7 L/min

QAI = QLVAD – QRVOT > QAI = 3.9 L/min – 1.7 L/min = 2.2 L/min

Selected References

1. Savage RM, Aronson S, Shernan SK. Comprehensive Textbook of Perioperative Transesophageal Echocardiography. 2nd ed. Wolters Kluwer; 2011.

2. Stainback RF, Estep JD, Agler DA, et al. Echocardiography in the management of patients with left ventricular assist devices: recommendations from the American Society of Echocardiography. J Am Soc Echocardiogr.2015;28(8):853-909.



3. An LVAD inflow cannula velocity that exceeds 4 m/s measured by continuous-wave Doppler in the immediate post-cardiopulmonary bypass period is most likely indicative of which of the following?


A. Outflow cannula obstruction by thrombus


B. Septal inflow cannula obstruction


C. Right ventricle failure


D. Hypovolemia

View Answer

3. Correct Answer: B. Septal inflow cannula obstruction

Rationale: A peak continuous-wave Doppler flow velocity of >2.3 m/s is highly suggestive of inflow cannula obstruction regardless of the manufacturer of the LVAD. In this case the peak velocity is much greater than 2.3 m/s; the only answer that suggests inflow cannula obstruction is answer B, which is septal inflow cannula obstruction. In the immediate postcardiopulmonary bypass period, left ventricle geometry can be altered depending on the insertion, with the potential for the inflow cannula to be directed toward the interventricular septum. With the inflow cannula directed at the interventricular septum, there is potential for inflow cannula obstruction and thus turbulent blood flow. Thrombus in the outflow graft generally does not give turbulent velocities as high as 4 m/s at the inflow cannula. Hypovolemia is possible, but suction events do not cause such turbulent flows through the LVAD inflow cannula. The velocity demonstrates a significant obstruction to LVAD inflow.

Selected References

1. Chumnanvej S, Wood MJ, MacGillivray TE, Videl Melo MF. Perioperative echocardiographic examination for ventricular assist device implantation. Anesth Analg. 2007 Sept;105(3):583-601.

2. Denalut AY, Couture P, Vegas A, Buithieu J, Tardif JC. Transesophageal Echocardiography Multimedia Manual a Perioperative Transdisciplinary Approach. Taylor & Francis Group, LLC; 2005.

3. Savage RM, Aronson S, Shernan SK. Comprehensive Textbook of Perioperative Transesophageal Echocardiography. 2nd ed. Wolters Kluwer; 2011.




4. Given the transthoracic echo image below (Figure 36.1).

This particular valvular pathology may occur in about 25% of LVAD patients, which of the following will increase the likelihood of this valvular pathology happening?







A. Moderate mitral valve regurgitation


B. Aortic valve that does not open


C. Presence of a bioprosthetic aortic valve


D. LV thrombus formation

View Answer

4. Correct Answer: B. Aortic valve that does not open

Rationale: New-onset aortic valve regurgitation will occur in about 25% to 33% of post-LVAD patients within 12 months of implantation. The largest risk factor for developing new-onset aortic valve regurgitation is the lack of opening of the aortic valve. The continuous negative pressure on the LV side of the aortic valve will lead to prolapse of one or multiple cusps, therefore leading to aortic valve regurgitation. It is unclear if the risk of aortic valve leaflet prolapse will be increased by the evidence of aortic valve thrombus during this time period. To minimize the development of aortic regurgitation in an aortic valve that does not open, many times the aortic valve will need to be sutured shut (Figure 36.8).






Selected References

1. Cowger J, Pagani FD, Haft JW, Romano MA, Aaronson KD, Kolias TJ. The development of aortic insufficiency in left ventricular assist device supported patients. Circ Heart Fail. 2010;3:668-674.

2. Pak SW, Uriel N, Takayama H, et al. Prevalence of de novo aortic insufficiency during long-term support with left ventricular assist devices. J Heart Lung Transplant. 2010;29:1172-1176.



5. Which of the following is not a reason to abort or stop a speed change (ramp) echocardiogram during a surveillance echocardiogram for mechanical support optimization 2 weeks post-LVAD implantation?


A. Suction event


B. Cannula flow reversal


C. Arrhythmia


D. Hypertension

View Answer

5. Correct Answer: C. Arrhythmia

Rationale: In the case of a medical optimization examination or speed change (ramp) echocardiography examination, there are certain factors that need to be evaluated. There should always be a medically trained mechanical support staff immediately available when mechanical support adjustments are made on LVAD patients. If no staff is available, the mechanical support and/or heart failure team should be notified that an optimization or speed change echocardiogram is being performed. It is necessary to outline and define the parameters that should be evaluated during the examination. The examination should be aborted if any of the following occur: (1) completion of the test as this is an obvious reason to stop the exam; (2) a suction event (at higher speeds); (3) new symptoms—including, but not limited to—palpitations, dizziness, chest pain, shortness of breath, or headache, which may be related to hypoperfusion or hypotension; (4) hypertension; and (5) cannula flow reversal. Hypertension (mean arterial pressure [MAP] > 85) can increase the risk of cerebral hemorrhage, stroke, renal failure, and thromboembolic events. Arrhythmias should not warrant a stop to the evaluation as these are very common post-LVAD implantation and generally do not cause the patient symptoms or the LVAD to malfunction.

Selected Reference

1. Stainback RF, Estep JD, Agler DA, et al. Echocardiography in the management of patients with left ventricular assist devices: recommendations from the American Society of Echocardiography. J Am Soc Echocardiogr.2015;28(8):853-909.



6. During insertion of an Impella 2.5 the characteristic color flow Doppler signal is entirely within the LV outflow tract without appreciable Doppler signal in the proximal aorta. What is the ideal positioning depth of the Impella device inflow port in regard to distance from the aortic valve annulus?


A. 4.0 to 4.5 cm


B. 2.5 to 3.0 cm


C. 3.0 to 3.5 cm


D. 3.5 to 4.0 cm

View Answer

6. Correct Answer: D. 3.5 to 4.0 cm

Rationale: During placement of an Impella 2.5 device the distance from the aortic valve annulus to the inflow port is ideally at a depth of 3.5 to 4.0 cm. Patient manipulation, movement, coughing, and transport have the potential to dislodge the Impella device out of the LVOT and into the proximal aorta if not properly secured after placement. Ideal placement of 3.5 to 4.0 cm from the aortic valve annulus should maintain the pump’s characteristic color flow Doppler signal distal to the aortic annulus.

Selected Reference

1. Stainback RF, Estep JD, Agler DA, et al. Echocardiography in the management of patients with left ventricular assist devices: recommendations from the American Society of Echocardiography. J Am Soc Echocardiogr.2015;28(8):853-909.



7. A 44-year-old male patient has been on venoarterial extracorporeal membrane oxygenation (VA-ECMO) with cardiogenic shock for 8 days and the surgical team feels that he is ready for decannulation. You are performing a transesophageal echocardiogram in the cardiovascular intensive care unit (CVICU) during a preoperative “turn down” evaluation. Which of the following parameters most strongly correlates with successful weaning from VA-ECMO with a VA-ECMO flow of <1.5 L/min?


A. Mitral valve E/A > 2.1


B. Aortic valve VTI > 10 cm


C. Mitral valve lateral annulus tissue Doppler <8 cm/s


D. Pulmonary capillary wedge pressure >25 mm Hg

View Answer

7. Correct Answer: B. Aortic valve VTI > 10 cm

Rationale: Controversy exists as to proper techniques to wean a patient from VA-ECMO. Current recommendations include TEE to guide weaning from VA-ECMO. Turn down studies help evaluate the extent of cardiac recovery from cardiogenic shock and improve the success of weaning by implementing a turn down TEE-driven protocol. When flows are <1.5 L/min, the three important parameters that should be evaluated are the aortic valve VTI >10 cm, the lateral mitral annulus peak systolic velocity >6 cm/s, and LVEF > 20%. Any of these three parameters correlate well with improved cardiac function. The only parameter in this question that correlates with suggested cardiac recovery and thus improved success from weaning from VA-ECMO is the aortic valve VTI >10 cm, the other answers show poor cardiac function and/or recovery with potentially elevated filling pressures, which would make weaning from VA-ECMO potentially more complicated.

Selected References

1. Aissaoui N, El-Banayosy A, Combes A. How to wean a patient from veno-arterial extracorporeal membrane oxygenation. Intensive Care Med. 2015;41:902-905.

2. Aissaoui N, Luyt C-E, Leprince P, et al. Predictors of successful extracorporeal membrane oxygenation (ECMO) weaning after assistance for refractory cardiogenic shock. Intensive Care Med. 2011;37:1738-1745.

3. Thomas TH, Price R, Ramaciotti C, Thompson M, Megison S, Lemler MS. Echocardiography, not chest radiography, for evaluation of cannula placement during pediatric extracorporeal membrane oxygenation. Pediatr Crit Care Med. 2009;10:56-59.



8. Which of the following is a good predictor of right ventricular failure following LVAD implantation?


A. Tricuspid annular plane systolic excursion of 16 mm


B. Pulmonary artery mean pressure of 15 mm Hg


C. LVEF of 10%


D. Right ventricular fractional area change (RVFAC) of 18%

View Answer

8. Correct Answer: D. Right ventricular fractional area change (RVFAC) of 18%

Rationale: A normal RVFAC is >35%. Many patients who are undergoing LVAD placement have some degree of RVD, whether it be mild, moderate, or severe. LVAD success is dependent on right ventricular function postimplantation. Assuming interventricular septal interdependence is not a contributing factor post-LVAD implantation, having a pre-LVAD RVFAC of <20% is associated with a significantly increased risk of right ventricular failure. With knowledge of preimplantation RVFAC and moderate-to-severe RVD, treatment with multiple inotropic agents, inhaled pulmonary vasodilators, and possible right ventricular mechanical support may be needed.

Selected References

1. Rudski LG, Lai WW, Afilalo J, et al. Guidelines for the echocardiographic assessment of the right heart in adults: a report from the American Society of Echocardiography endorsed by the European Association of Echocardiography, a registered branch of the European Society of Cardiology, and the Canadian Society of Echocardiography. J Am Soc Echocardiogr. 2010 Jul;23(7):685-713.

2. Scalia GM, McCarthy PM, Savage RM, Smedira NG, Thomas JD. Clinical utility of echocardiography in the management of implantable ventricular assist devices. J Am Soc Echocardiogr. 2000;13(8):754-763.




9. A 26-year-old female patient with postpartum cardiomyopathy and cardiogenic shock is 7 days post implantation of an LVAD. She remains intubated and has been hypotensive requiring continuous inotropic support and now has multiple pulsatility index events and low cardiac output alarms with patient repositioning and deep breathing. Given the transesophageal echocardiography (TEE) findings in Table 36.1, which of the following is the most likely cause of the LVAD malfunction?









A. Hypotension


B. Mitral valve regurgitation


C. Hypovolemia


D. Pulmonic valve regurgitation

View Answer

9. Correct Answer: C. Hypovolemia

Rationale: Hypovolemia in LVAD patients is generally a clinical diagnosis with a low CVP, hypotension, and increase in pulsatility events. However, focused TEE in the ICU can help guide the cause of low cardiac output and increased event alarms with direct visualization of the cardiac function. A focused TEE evaluation is vital to rule out cardiac tamponade, right ventricular failure, and inflow cannula obstruction. Given the data in the question are suggestive of low preload and low LV volume status, combined with device alarms and low cardiac output, this is highly suggestive of hypovolemia. This patient has low filling pressures on the right side of the heart combined with normal right ventricular function. With the LVAD inflow cannula velocity relatively normal, it means the device is not occluded or rather there is no evidence of inflow cannula obstruction. However, in this question when the patient moves or initiates a large deep breath, the low cardiac output alarm goes off because of relative obstruction due to hypovolemia, causing a relative positional and functional obstruction of the inflow cannula.

Selected References

1. Churnnanvej S, Wood MJ, MacGillivray TE, Melo ME. Perioperative echocardiographic examination for ventricular assist device implantation. Anesth Analg. 2007;105(3):583-601.

2. Horton SC, Khodaverdian R, Chatelain P, et al. Left ventricular assist device malfunction: an approach to diagnosis by echocardiography. Am Coil Cardiol. 2005;45(9):1435-1440.

3. Szymanski P, Religa G, Klisiewicz A, Baranska K, Hoffman P. Diagnosis of biventricular assist device inflow cannula obstruction. Echocardiography. 2007;24(4):420-424.



10. A 51-year-old male patient in cardiogenic shock on high-dose vasopressors and VA-ECMO has just returned from the cardiac catheterization laboratory after Impella 2.5 insertion for LV decompression. The patient remains unstable and a bedside focused transthoracic echocardiogram (TTE) image is shown in Figure 36.2.






What is the next best step in management?


A. Nothing—the Impella is in proper position


B. Advance the Impella another 3 cm into the LV outflow tract (LVOT)


C. Reposition the Impella as it is potentially obstructing mitral valve inflow


D. Pull the Impella back 2.5 cm as it is too deep into the left ventricle

View Answer

10. Correct Answer: C. Reposition the Impella as it is potentially obstructing mitral valve inflow

Rationale: Figure 36.9 shows the apical three-chamber view. TTE demonstrates that the Impella is not properly positioned with its indicator toward the apex. In fact, looking at the image shows that the Impella is almost abutting the inferolateral wall. Its pigtail is not well visualized and not easily visualized, but based on the course of the Impella through the aortic valve, it can be assumed that it is physically in contact with the inferolateral wall. With the course of the Impella toward the inferolateral wall and in such close proximity to the anterior mitral valve leaflet, there may be concern for mitral valve obstruction caused by the Impella. Based on Figure 36.9 it appears as if the Impella may be restricting the opening of the mitral valve as the course of the Impella device is in very close proximity to the anterior leaflet of the mitral valve contributing to the hemodynamic instability.






Selected References

1. Savage RM, Aronson S, Shernan SK. Comprehensive Textbook of Perioperative Transesophageal Echocardiography. 2nd ed. Wolters Kluwer; 2011.

2. Stainback RF, Estep JD, Agler DA, et al. Echocardiography in the management of patients with left ventricular assist devices: recommendations from the American Society of Echocardiography. J Am Soc Echocardiogr.2015;28(8):853-909.



11. After placement of an Impella 2.5 via the right femoral artery in the catheterization lab, the patient is transported to the cardiothoracic ICU (CTICU). During transport the patient becomes hemodynamically unstable and an emergent TEE is performed immediately upon arrival in the CTICU (Figure 36.3).







Based on the TEE in Figure 36.3, what is the next best step in the management of this patient?

Jun 9, 2022 | Posted by in CARDIOLOGY | Comments Off on Mechanical Circulatory Support

Full access? Get Clinical Tree

Get Clinical Tree app for offline access